2011 AMC 8 Problems/Problem 21

Revision as of 19:14, 4 November 2012 by Mathway (talk | contribs)

Problem

Students guess that Norb's age is $24, 28, 30, 32, 36, 38, 41, 44, 47$, and $49$. Norb says, "At least half of you guessed too low, two of you are off by one, and my age is a prime number." How old is Norb?

$\textbf{(A) }29\qquad\textbf{(B) }31\qquad\textbf{(C) }37\qquad\textbf{(D) }43\qquad\textbf{(E) }48$

Solution

If at least half the guesses are two low, then his age must be greater than $36.$

If two of the guesses are off by one, then his age is in between two guesses whose difference is $2.$ It could $31,37,$ or $48,$ but because it is greater than $36$ it can only be $37$ or $48.$

Lastly, his age is a prime number so the answer must be $\boxed{\textbf{(C)}\ 37}$

See Also

2011 AMC 8 (ProblemsAnswer KeyResources)
Preceded by
Problem 20
Followed by
Problem 22
1 2 3 4 5 6 7 8 9 10 11 12 13 14 15 16 17 18 19 20 21 22 23 24 25
All AJHSME/AMC 8 Problems and Solutions